1
$\begingroup$

Let $U_1,U_2,U_3,\dots$ be IID uniform on $[0,1]$. For each $n\geq 1$ let $$U_{1:n}<U_{2:n}<\dots<U_{n:n}$$ be the order statistic of $(U_1,\dots,U_n)$. Independent of the $U$ process there is a standard polya urn process $S=(S_n)_{n\geq 1}$ with distribution specified by $$(S_n)_{n\geq 1}~~\sim~~\big(1+\sum_{j=1}^{n-1} 1(V_j\leq V_0)\big)_{n\geq 1},$$ where $V_0,V_1,V_2,\dots$ are IID uniform on $[0,1]$. So it is clear that $\frac{1}{n}S_n$ converges alsmost surely towards some $V_0$ and $V_0$ is uniform on $[0,1]$.

Now let $B\subseteq [0,1]$ be any Borel set and $1_B(\cdot)$ the indicator function of $B$. Question: $$\text{Does $1_B(U_{S_n:n})$ converge almost surely as $n\rightarrow\infty$ towards $1_B(V_0)$?}$$

It is clear that $U_{S_n:n}$ tends to $V_0$ almost surely. Since the random variables in the question are $\{0,1\}$-valued, a.s. convergence means that they stay finally constant a.s..

Remark: This question is about some kind of '0-1'- representations of Borel sets. For each Borel set $B\subseteq [0,1]$ and $n\geq 1$ I'm interested in the $\{0,1\}$-string $$\big[1_B(U_{1:n}),1_B(U_{2:n}),\dots,1_B(U_{n:n})\big]~\in\{0,1\}^n.$$ For large $n$ these strings should 'look like' the set $B$ (modulo uniform distribution). What can one say about long substrings $11\dots 11$ for large $n$?

$\endgroup$
1
  • $\begingroup$ Seems like a job for the Lebesgue density theorem... $\endgroup$ Jun 28, 2016 at 13:42

1 Answer 1

3
$\begingroup$

Suppose that $U_n,V$ are some random variables such that $V$ is uniformly distributed on $[0,1]$, $U_n$ converges to $V$ a.s. and for every $n$, $P(U_n\ne V)=1$ (To be honest, I got a bit confused trying to read your definitions, but it certainly looks like no fixed variable can be exactly equal to the limiting one with positive probability). Then there exists a Borel set $B\subset [0,1]$ such that $1_B(U_n)$ fails to converge to $1_B(V)$ a.s. The construction is pretty straightforward. First, create sets $A_n$ of measure about $\mu_n$ such that $P(\text{there is }k>n\text{ such that }U_k\in A_n)>1/2$ (just split $[0,1]$ into short intervals of length $\ell$ and choose each one independently with probability $\mu_n$. Note that for fixed $\omega$, the events $U_k(\omega)\in A_n$ and $U_m(\omega)\in A_n$ are independent if $|U_k(\omega)-U_m(\omega)|>\ell$ and there are infinitely many different values $U_k(\omega)$ for a.e. $\omega$). Now just take $B=\cup A_n$ where $\mu_n$ are chosen so that $\sum_n\mu_n\le 1/4$. Of course, the first two assumptions can be dropped, but then you have to do some casework and the argument gets longer.

$\endgroup$
2
  • $\begingroup$ Thank's a lot for your answers! I understand the other answer, but I have some troubles with this one...First of all: Yes, no fixed vairable is equal to the limiting one a.s. I have some trouble handling the 'random-set-construction' of the counter example...Why is $P(\text{there is $k>n$ such that $U_k\in A_n$})>1/2$? $\endgroup$
    – user240643
    Sep 2, 2016 at 15:31
  • $\begingroup$ @user240643 If you have plenty of independent random events of a fixed probability (however small), then the probability that at least one happens is close to $1$. So, just choose $\ell$ so small that for the vast majority of $\omega\in\Omega$ you can find $1/\mu_n$ or so $\ell$-separated values among $U_k(\omega)$ with $k\ge n$. $\endgroup$
    – fedja
    Sep 2, 2016 at 16:49

Your Answer

By clicking “Post Your Answer”, you agree to our terms of service and acknowledge you have read our privacy policy.

Not the answer you're looking for? Browse other questions tagged or ask your own question.